Bất phương trình logarit

Kỳ thi ĐGNL ĐHQG Hà Nội

Đổi lựa chọn

Câu 1 Trắc nghiệm

Bất phương trình  \(\log_{{\frac{4}{{25}}}}(x + 1) \ge \log_{{\frac{2}{5}}}x\) tương đương với bất phương trình nào dưới đây?

Bạn đã chọn sai | Đáp án đúng: c
Bạn đã chọn đúng | Đáp án đúng: c
Bạn chưa làm câu này | Đáp án đúng: c

Ta có ${\log _{\frac{4}{{25}}}}\left( {x + 1} \right) = \dfrac{1}{2}{\log _{\frac{2}{5}}}\left( {x + 1} \right)$ nên bất phương trình đã cho tương đương với:

$\dfrac{1}{2}{\log _{\frac{2}{5}}}\left( {x + 1} \right) \ge {\log _{\frac{2}{5}}}x \Leftrightarrow {\log _{\frac{2}{5}}}\left( {x + 1} \right) \ge 2{\log _{\frac{2}{5}}}x$

Câu 2 Trắc nghiệm

Giải bất phương trình $\log_{2}\left( {3x-1} \right) \ge 3$.

Bạn đã chọn sai | Đáp án đúng: a
Bạn đã chọn đúng | Đáp án đúng: a
Bạn chưa làm câu này | Đáp án đúng: a

Điều kiện: $x > \dfrac{1}{3}$

BPT $ \Leftrightarrow 3x - 1 \ge 8 \Leftrightarrow x \ge 3$

Kết hợp điều kiện ta được $x \ge 3$

Câu 3 Trắc nghiệm

Giải bất phương trình \({\log _{\frac{1}{3}}}(x + {9^{500}}) >  - 1000\)

Bạn đã chọn sai | Đáp án đúng: d
Bạn đã chọn đúng | Đáp án đúng: d
Bạn chưa làm câu này | Đáp án đúng: d

Điều kiện $x + {9^{500}} > 0 \Leftrightarrow x >  - {9^{500}}$

Vì ${\rm{0 < a}} = \dfrac{1}{3} < 1$ nên

$\begin{array}{l}{\log _{\dfrac{1}{3}}}\left( {x + {9^{500}}} \right) >  - 1000 \Leftrightarrow 0 < x + {9^{500}} < {\left( {\dfrac{1}{3}} \right)^{ - 1000}} \Leftrightarrow 0 < x + {9^{500}} < {3^{1000}}\\ \Leftrightarrow  - {9^{500}} < x < {3^{1000}} - {9^{500}} \Leftrightarrow  - {3^{1000}} < x < {3^{1000}} - {3^{1000}} \Leftrightarrow  - {3^{1000}} < x < 0\end{array}$

Câu 4 Trắc nghiệm

Số nguyên nhỏ nhất thỏa mãn $\log_{2}\left( {5x-3} \right) > 5$ là:

Bạn đã chọn sai | Đáp án đúng: b
Bạn đã chọn đúng | Đáp án đúng: b
Bạn chưa làm câu này | Đáp án đúng: b

Điều kiện: $x > \dfrac{3}{5}$

${\log _2}\left( {5x - 3} \right) > 5 \Leftrightarrow 5x - 3 > {2^5} \Leftrightarrow 5x > 35 \Leftrightarrow x > 7$

Vậy số nguyên nhỏ nhất thỏa mãn bất phương trình là \(x = 8\).

Câu 5 Trắc nghiệm

Cho $m = {\log _a}\sqrt {ab} $ với $a,b > 1$ và $P = \log _a^2b + 54{\log _b}a$. Khi đó giá trị của $m$ để $P$ đạt giá trị nhỏ nhất là:

Bạn đã chọn sai | Đáp án đúng: a
Bạn đã chọn đúng | Đáp án đúng: a
Bạn chưa làm câu này | Đáp án đúng: a

Ta có $P = \log _a^2b + 54{\log _b}a = \log _a^2b + \dfrac{{54}}{{{{\log }_a}b}}$

Đặt $t = {\log _a}b$ thì $P = {t^2} + \dfrac{{54}}{t}$

Vì \(a,\,\,b > 1\) nên $t = {\log _a}b > 0$.

Áp dụng bất đẳng thức Cô – si ta có

$P = {t^2} + \dfrac{{54}}{t} = {t^2} + \dfrac{{27}}{t} + \dfrac{{27}}{t} \ge 3\sqrt[3]{{{{27}^2}}} = 27.$

Đẳng thức xảy ra khi và chỉ khi ${t^2} = \dfrac{{27}}{t} \Leftrightarrow t = 3.$

Ta có $m = {\log _a}\sqrt {ab}  = \dfrac{1}{2}{\log _a}\left( {ab} \right)$$ = \dfrac{1}{2}\left( {1 + {{\log }_a}b} \right) = \dfrac{1}{2}\left( {1 + t} \right) = \dfrac{1}{2}\left( {1 + 3} \right) = 2$

Câu 6 Trắc nghiệm

Tìm tập nghiệm \(S\) của bất phương trình \({\log _{\frac{1}{2}}}\left( {x - 1} \right) > {\log _{\frac{1}{2}}}\left( {5 - 2x} \right)\).

Bạn đã chọn sai | Đáp án đúng: d
Bạn đã chọn đúng | Đáp án đúng: d
Bạn chưa làm câu này | Đáp án đúng: d

Điều kiện \(\left\{ \begin{array}{l}x - 1 > 0\\5 - 2x > 0\end{array} \right. \Leftrightarrow \left\{ \begin{array}{l}x > 1\\x < \dfrac{5}{2}\end{array} \right.\)

\({\log _{\frac{1}{2}}}\left( {x - 1} \right) > {\log _{\frac{1}{2}}}\left( {5 - 2x} \right) \Leftrightarrow x - 1 < 5 - 2x \Leftrightarrow x < 2\).

Kết hợp với điều kiện suy ra $S= (1;2)$.  

Câu 7 Trắc nghiệm

Tìm tất cả các giá trị thực của tham số \(m\) để bất phương trình \(4.{\left( {{{\log }_2}\sqrt x } \right)^2} + {\log _2}x + m \ge 0\) nghiệm đúng với mọi giá trị \(x \in \left[ {1;64} \right]\).

Bạn đã chọn sai | Đáp án đúng: c
Bạn đã chọn đúng | Đáp án đúng: c
Bạn chưa làm câu này | Đáp án đúng: c

Điều kiện : $x > 0$

\(4.{\left( {{{\log }_2}\sqrt x } \right)^2} + {\log _2}x + m \ge 0 \Leftrightarrow 4.{\left( {{{\log }_2}\sqrt x } \right)^2} + 2.{\log _2}\sqrt x  \ge  - m\)(1)

Đặt \(t = {\log _2}\sqrt x \). Khi \(x \in \left[ {1;64} \right] \Rightarrow t \in \left[ {0;3} \right]\).

Ta có bất phương trình \(4{t^2} + 2t \ge  - m\).

Xét \(f(t) = 4{t^2} + 2t;f'(t) = 8t + 2 > 0,\forall t \in \left[ {0;3} \right]\)

Để (1) nghiệm đúng với \(\forall t \in \left[ {0;3} \right]\) thì $\mathop {\min }\limits_{\left[ {0;3} \right]} f\left( t \right) \ge  - m$

\( \Leftrightarrow f(0) \ge  - m \Leftrightarrow 0 \ge  - m \Leftrightarrow m \ge 0\).

Câu 8 Trắc nghiệm

Xét bất phương trình \(\log _2^22x - 2\left( {m + 1} \right){\log _2}x - 2 < 0\). Tìm tất cả các giá trị của tham số \(m\) để bất phương trình có nghiệm thuộc khoảng \(\left( {\sqrt {2;}  + \infty } \right)\).

Bạn đã chọn sai | Đáp án đúng: c
Bạn đã chọn đúng | Đáp án đúng: c
Bạn chưa làm câu này | Đáp án đúng: c

Điều kiện: \(x > 0\)

\(\log _2^2 2x - 2\left( {m + 1} \right){\log _2}x - 2 < 0\)

\( \Leftrightarrow {\left( {1 + {{\log }_2}x} \right)^2} - 2\left( {m + 1} \right){\log _2}x - 2 < 0{\rm{   }}\left( 1 \right)\).

Đặt \(t = {\log _2}x\). Vì $x > \sqrt 2 $ nên ${\log _2}x > {\log _2}\sqrt 2  = \dfrac{1}{2}$.

Do đó $t \in \left( {\dfrac{1}{2}; + \infty } \right)$

\(\left( 1 \right)\) thành \({\left( {1 + t} \right)^2} - 2\left( {m + 1} \right)t - 2 < 0\)\( \Leftrightarrow {t^2} - 2mt - 1 < 0\) \(\left( 2 \right)\)

Yêu cầu bài toán tương đương tìm \(m\) để bpt $(2)$ có nghiệm thuộc \(\left( {\dfrac{1}{2}; + \infty } \right)\).

Xét bất phương trình $(2)$ có: $\Delta ' = {m^2} + 1 > 0,{\rm{ }}\forall m \in \mathbb{R}$.

\(f\left( t \right) = {t^2} - 2mt - 1 = 0\) có \(ac < 0\) nên $f(t)$ luôn có $2$ nghiệm phân biệt \({t_1} < 0 < {t_2}\) nên tập nghiệm của $(2)$ là $(t_1;t_2)$

Khi đó cần \(\dfrac{1}{2} < {t_2} \Leftrightarrow m + \sqrt {{m^2} + 1}  > \dfrac{1}{2} \Leftrightarrow m >  - \dfrac{3}{4}\)

Câu 9 Trắc nghiệm

Tập nghiệm của bất phương trình $\ln\left[ {\left( {x - 1} \right)\left( {x - 2} \right)\left( {x - 3} \right) + 1} \right] > 0$ là:

Bạn đã chọn sai | Đáp án đúng: a
Bạn đã chọn đúng | Đáp án đúng: a
Bạn chưa làm câu này | Đáp án đúng: a

$\begin{array}{l}\ln \left[ {\left( {x - 1} \right)\left( {x - 2} \right)\left( {x - 3} \right) + 1} \right] > 0 \Leftrightarrow \left( {x - 1} \right)\left( {x - 2} \right)\left( {x - 3} \right) + 1 > 1\\ \Leftrightarrow \left( {x - 1} \right)\left( {x - 2} \right)\left( {x - 3} \right) > 0\end{array}$

$ \Rightarrow x \in (1;2) \cup (3; + \infty )$

Câu 10 Trắc nghiệm

Một người tham gia chương trình bảo hiểm An sinh xã hội của công ty Bảo Việt với thể lệ như sau: Cứ đến tháng $9$ hàng năm người đó đóng vào công ty là $12$ triệu đồng với lãi suất hàng năm không đổi là $6\% $ / năm. Hỏi sau đúng $18$ năm kể từ ngày đóng, người đó thu về được tất cả bao nhiêu tiền? Kết quả làm tròn đến hai chữ số phần thập phân.

Bạn đã chọn sai | Đáp án đúng: d
Bạn đã chọn đúng | Đáp án đúng: d
Bạn chưa làm câu này | Đáp án đúng: d

Gọi số tiền đóng hàng năm là $A = 12$ (triệu đồng), lãi suất là $r = 6\%  = 0,06$.

Sau \(1\) năm, nếu người đó đi rút tiền thì sẽ nhận được số tiền là \({A_1} = A\left( {1 + r} \right)\). (nhưng người đó không rút mà lại đóng thêm $A$ triệu đồng nữa, nên số tiền gốc để tính lãi năm sau là \({A_1} + A\)).

Sau \(2\) năm, nếu người đó đi rút tiền thì sẽ nhận được số tiền là:

\({A_2} = \left( {{A_1} + A} \right)\left( {1 + r} \right) = \left[ {A\left( {1 + r} \right) + A} \right]\left( {1 + r} \right) = A{\left( {1 + r} \right)^2} + A\left( {1 + r} \right)\)

Sau \(3\) năm, nếu người đó đi rút tiền thì sẽ nhận được số tiền là:

\({A_3} = \left( {{A_2} + A} \right)\left( {1 + r} \right) = \left[ {A{{\left( {1 + r} \right)}^2} + A\left( {1 + r} \right) + A} \right]\left( {1 + r} \right) = A{\left( {1 + r} \right)^3} + A{\left( {1 + r} \right)^2} + A\left( {1 + r} \right)\)

Sau \(18\) năm, người đó đi rút tiền thì sẽ nhận được số tiền là:

${A_{18}} = A{\left( {1 + r} \right)^{18}} + A{\left( {1 + r} \right)^{17}} + ... + A{\left( {1 + r} \right)^2} + A\left( {1 + r} \right)$

Tính: ${A_{18}} = A\left[ {{{\left( {1 + r} \right)}^{18}} + {{\left( {1 + r} \right)}^{17}} + ... + {{\left( {1 + r} \right)}^2} + \left( {1 + r} \right) + 1 - 1} \right]$

$ \Rightarrow {A_{18}} = A\left[ {\dfrac{{{{\left( {1 + r} \right)}^{19}} - 1}}{{\left( {1 + r} \right) - 1}} - 1} \right] = A\left[ {\dfrac{{{{\left( {1 + r} \right)}^{19}} - 1}}{r} - 1} \right] = 12\left[ {\dfrac{{{{\left( {1 + 0,06} \right)}^{19}} - 1}}{{0,06}} - 1} \right] \approx 393,12$

Câu 11 Trắc nghiệm

Tập nghiệm của bất phương trình $\log\left( {{x^2} + 25} \right) > \log\left( {10x} \right)$ là:

Bạn đã chọn sai | Đáp án đúng: b
Bạn đã chọn đúng | Đáp án đúng: b
Bạn chưa làm câu này | Đáp án đúng: b

Điều kiện: $x > 0$

$\log ({x^2} + 25) > \log (10x) \Leftrightarrow {x^2} + 25 > 10x \Leftrightarrow {(x - 5)^2} > 0 \Leftrightarrow x \ne 5$

Tập nghiệm của bất phương trình là: $(0;5) \cup (5; + \infty )$

Câu 12 Trắc nghiệm

Cho hai số thực $a$, $b$ thỏa mãn $a > b > \dfrac{4}{3}$ và biểu thức $P = 16{\log _a}\left( {\dfrac{{{a^3}}}{{12b - 16}}} \right) + 3\log _{\frac{a}{b}}^2a$ có giá trị nhỏ nhất. Tính $a + b.$

Bạn đã chọn sai | Đáp án đúng: d
Bạn đã chọn đúng | Đáp án đúng: d
Bạn chưa làm câu này | Đáp án đúng: d

Ta có: $P = 16{\log _a}\left( {\dfrac{{{a^3}}}{{12b - 16}}} \right) + 3\log _{\frac{a}{b}}^2a$. Vì số hạng thứ hai chứa ${\log _{\frac{a}{b}}}a$ nên ta cố gắng đưa ${\log _a}\left( {\dfrac{{{a^3}}}{{12b - 16}}} \right)$ về ${\log _a}\dfrac{a}{b}$. Điều này buộc ta cần đánh giá $12b - 16 \le {b^3}$. Thật vậy:

Ta có: $12b - 16 \le {b^3} \Leftrightarrow {\left( {b - 2} \right)^2}\left( {b + 4} \right) \ge 0$ (Đúng).

Suy ra: $\dfrac{{{a^3}}}{{12b - 16}} \ge \dfrac{a}{b} > 1$ $ \Rightarrow {\log _a}\left( {\dfrac{{{a^3}}}{{12b - 16}}} \right) \ge {\log _a}{\left( {\dfrac{a}{b}} \right)^3} = 3{\log _a}\dfrac{a}{b} > 0$

Do đó:

$P \ge 48{\log _a}\dfrac{a}{b} + 3\log _{\frac{a}{b}}^2a$ $ = 3\left( {8{{\log }_a}\dfrac{a}{b} + 8{{\log }_a}\dfrac{a}{b} + \log _{\frac{a}{b}}^2a} \right)$

Áp dụng bất đẳng thức Cauchy cho $3$ số dương $8{\log _a}\dfrac{a}{b}$, $8{\log _a}\dfrac{a}{b}$, $\log _{\frac{a}{b}}^2a$ ta được:

$P \ge 3 \cdot 3 \cdot \sqrt[3]{{\left( {8{{\log }_a}\dfrac{a}{b} \cdot 8{{\log }_a}\dfrac{a}{b} \cdot \log _{\frac{a}{b}}^2a} \right)}} = 9\sqrt[3]{{64}} = 36.$

Đẳng thức xảy ra khi và chỉ khi

$\left\{ \begin{array}{l}b = 2\\8{\log _a}\dfrac{a}{b} = \log _{\frac{a}{b}}^2a = 4\end{array} \right. \Leftrightarrow \left\{ \begin{array}{l}b = 2\\{\log _a}\dfrac{a}{b} = \dfrac{1}{2}\end{array} \right. \Leftrightarrow \left\{ \begin{array}{l}b = 2\\{\log _a}2 = \dfrac{1}{2}\end{array} \right. \Leftrightarrow \left\{ \begin{array}{l}b = 2\\a = 4\end{array} \right..$

Vậy $a + b = 6.$

Câu 13 Trắc nghiệm

Tập nghiệm của bất phương trình $({2^{{x^2} - 4}} - 1).\ln {x^2} < 0$ là:

Bạn đã chọn sai | Đáp án đúng: b
Bạn đã chọn đúng | Đáp án đúng: b
Bạn chưa làm câu này | Đáp án đúng: b

Điều kiện: \(x \ne 0\).

\(\begin{array}{l}({2^{{x^2} - 4}} - 1) \ln{x^2} < 0 \Rightarrow \left[ \begin{array}{l}\left\{ \begin{array}{l}({2^{{x^2} - 4}} - 1) > 0\\ \ln{x^2} < 0\end{array} \right.\\\left\{ \begin{array}{l}({2^{{x^2} - 4}} - 1) < 0\\ \ln{x^2} > 0\end{array} \right.\end{array} \right. \Leftrightarrow \left[ \begin{array}{l}\left\{ \begin{array}{l}{2^{{x^2} - 4}} > 1\\{x^2} < 1\end{array} \right.\\\left\{ \begin{array}{l}{2^{{x^2} - 4}} < 1\\{x^2} > 1\end{array} \right.\end{array} \right. \Leftrightarrow \left[ \begin{array}{l}\left\{ \begin{array}{l}{x^2} - 4 > 0\\{x^2} < 1\end{array} \right.\\\left\{ \begin{array}{l}{x^2} - 4 < 0\\{x^2} > 1\end{array} \right.\end{array} \right.\\ \Leftrightarrow \left[ \begin{array}{l}\left\{ \begin{array}{l}x > 2;x <  - 2\\ - 1 < x < 1\end{array} \right.\\\left\{ \begin{array}{l} - 2 < x < 2\\x > 1;x <  - 1\end{array} \right.\end{array} \right. \Leftrightarrow \left[ \begin{array}{l} - 2 < x <  - 1\\1 < x < 2\end{array} \right. \Rightarrow x \in \left( { - 2; - 1} \right) \cup \left( {1;2} \right)\end{array}\)

Câu 14 Trắc nghiệm

Tập hợp nghiệm của bất phương trình ${\log _{\frac{1}{3}}}\left( {{x^2} - 2x + 1} \right) < {\log _{\frac{1}{3}}}\left( {x - 1} \right)$ là:

Bạn đã chọn sai | Đáp án đúng: c
Bạn đã chọn đúng | Đáp án đúng: c
Bạn chưa làm câu này | Đáp án đúng: c

Điều kiện: $\left\{ \begin{array}{l}{x^2} - 2x + 1 > 0\\x - 1 > 0\end{array} \right. \Leftrightarrow \left\{ \begin{array}{l}{\left( {x - 1} \right)^2} > 0\\x - 1 > 0\end{array} \right. \Leftrightarrow x > 1$

${\log _{\dfrac{1}{3}}}({x^2} - 2x + 1) < {\log _{\dfrac{1}{3}}}(x - 1) \Leftrightarrow {x^2} - 2x + 1 > x - 1 > 0$

$ \Leftrightarrow \left\{ \begin{array}{l}{x^2} - 3x + 2 > 0\\x - 1 > 0\end{array} \right. \Leftrightarrow \left\{ \begin{array}{l}\left( {x - 1} \right)(x - 2) > 0\\x - 1 > 0\end{array} \right. \Leftrightarrow x > 2$

Câu 15 Trắc nghiệm

Nghiệm của bất phương trình ${\log _2}(x + 1) + {\log _{\frac{1}{2}}}\sqrt {x + 1}  \le 0$ là :

Bạn đã chọn sai | Đáp án đúng: b
Bạn đã chọn đúng | Đáp án đúng: b
Bạn chưa làm câu này | Đáp án đúng: b

Điều kiện $x > -1$.

Khi đó ta có:

$\begin{array}{l}{\log _2}(x + 1) - lo{g_2}\sqrt {x + 1}  \le 0 \Leftrightarrow {\log _2}\dfrac{{x + 1}}{{\sqrt {x + 1} }} \le 0 \Leftrightarrow \dfrac{{x + 1}}{{\sqrt {x + 1} }} \le 1\\ \Leftrightarrow \dfrac{{{{(\sqrt {x + 1} )}^2}}}{{\sqrt {x + 1} }} \le 1 \Leftrightarrow \sqrt {x + 1}  \le 1 \Leftrightarrow x \le 0\end{array}$

Kết hợp với điều kiện ta được: $ - 1 < x \le 0$

Câu 16 Trắc nghiệm

Giải bất phương trình \({\log _{0,7}}\left( {{{\log }_6}\dfrac{{{x^2} + x}}{{x + 4}}} \right) < 0\)

Bạn đã chọn sai | Đáp án đúng: a
Bạn đã chọn đúng | Đáp án đúng: a
Bạn chưa làm câu này | Đáp án đúng: a

${\log _{0,7}}({\log _6}\dfrac{{{x^2} + x}}{{x + 4}}) < 0$ .

Đkxđ: $\left\{ \begin{array}{l}{\log _6}\dfrac{{{x^2} + x}}{{x + 4}} > 0\\\dfrac{{{x^2} + x}}{{x + 4}} > 0\end{array} \right. \Leftrightarrow \left[ \begin{array}{l} - 4 < x <  - 2\\x > 2\end{array} \right.(*)$

\(\begin{array}{l}{\log _6}\dfrac{{{x^2} + x}}{{x + 4}} > 0,{7^0} = 1 \Leftrightarrow \dfrac{{{x^2} + x}}{{x + 4}} > 6 \Leftrightarrow \dfrac{{{x^2} + x}}{{x + 4}} - 6 > 0\\ \Leftrightarrow \dfrac{{{x^2} - 5{\rm{x}} - 24}}{{x + 4}} > 0 \Leftrightarrow \dfrac{{(x - 8)(x + 3)}}{{x + 4}} > 0\end{array}\)

Xét dấu \(f\left( x \right) = \dfrac{{(x - 8)(x + 3)}}{{x + 4}}\):

Vậy \( - 4 < x <  - 3\) hoặc \(x > 8\).

Kết hợp với điều kiện ta được \( - 4 < x <  - 3\) hoặc \(x > 8\).

Câu 17 Trắc nghiệm

Tìm tập hợp nghiệm $S$ của bất phương trình: \({\log _{\frac{\pi }{4}}}({x^2} + 1) < {\log _{\frac{\pi }{4}}}(2x + 4)\)

Bạn đã chọn sai | Đáp án đúng: c
Bạn đã chọn đúng | Đáp án đúng: c
Bạn chưa làm câu này | Đáp án đúng: c

Điều kiện $x>-2$

Bất phương trình \( \Leftrightarrow {x^2} + 1 > 2x + 4\,(do\,\dfrac{\pi }{4} < 1)\) \( \Leftrightarrow {x^2} - 2x - 3 = (x + 1)(x - 3) > 0\)

Nên $x>3$ hoặc $x<-1$.

Kết hợp điều kiện $x>-2$ ta được $x>3$ hoặc $-2<x<-1$.

Câu 18 Trắc nghiệm

Giải bất phương trình \({\log _3}({2^x} - 3) < 0\)

Bạn đã chọn sai | Đáp án đúng: c
Bạn đã chọn đúng | Đáp án đúng: c
Bạn chưa làm câu này | Đáp án đúng: c

Bất phương trình tương đương:

$\left\{ \begin{array}{l}{2^x} - 3 > 0\\{2^x} - 3 < 1\end{array} \right. \Leftrightarrow \left\{ \begin{array}{l}x > {\log _2}3\\x < 2\end{array} \right. \Leftrightarrow {\log _2}3 < x < 2$.

Câu 19 Trắc nghiệm

Xét các số thực \(a\), \(b\) thỏa mãn điều kiện \(\dfrac{1}{3} < b < a < 1\). Tìm giá trị nhỏ nhất của biểu thức $P = {\log _a}\left( {\dfrac{{3b - 1}}{4}} \right) + 12\log _{\frac{b}{a}}^2a - 3$.

Bạn đã chọn sai | Đáp án đúng: c
Bạn đã chọn đúng | Đáp án đúng: c
Bạn chưa làm câu này | Đáp án đúng: c

$P = {\log _a}\left( {\dfrac{{3b - 1}}{4}} \right) + 12{\left( {{{\log }_{\frac{b}{a}}}a} \right)^2} - 3$$ = {\log _a}\left( {\dfrac{{3b - 1}}{4}} \right) + 12{\left( {\dfrac{1}{{{{\log }_a}\dfrac{b}{a}}}} \right)^2} - 3$$ = {\log _a}\left( {\dfrac{{3b - 1}}{4}} \right) + 12{\left( {\dfrac{1}{{{{\log }_a}b - 1}}} \right)^2} - 3$

Ta có: $\dfrac{{3b - 1}}{4} \le {b^3}$$ \Leftrightarrow 3b - 1 \le 4{b^3}$$ \Leftrightarrow 4{b^3} - 3b + 1 \ge 0$$ \Leftrightarrow \left( {b + 1} \right)\left( {4{b^2} - 4b + 1} \right) \ge 0$

$ \Leftrightarrow \left( {b + 1} \right){\left( {2b - 1} \right)^2} \ge 0$ (luôn đúng với \(\dfrac{1}{3} < b < 1\))

$ \Rightarrow {\log _a}\left( {\dfrac{{3b - 1}}{4}} \right) \ge {\log _a}{b^3}$ ( vì \(a < 1\)) $ \Rightarrow {\log _a}\left( {\dfrac{{3b - 1}}{4}} \right) \ge 3{\log _a}b$

Do đó $P \ge 3{\log _a}b + \dfrac{{12}}{{{{\left( {{{\log }_a}b - 1} \right)}^2}}} - 3$$ \Leftrightarrow P \ge 3\left( {{{\log }_a}b - 1} \right) + \dfrac{{12}}{{{{\left( {{{\log }_a}b - 1} \right)}^2}}}$ \(\left( * \right)\)

Vì \(\dfrac{1}{3} < b < a < 1\) nên \({\log _a}b > 1\)

Áp dụng bất đẳng thức Côsi cho \(3\) số dương: \(\dfrac{3}{2}\left( {{{\log }_a}b - 1} \right)\), \(\dfrac{3}{2}\left( {{{\log }_a}b - 1} \right)\), $\dfrac{{12}}{{{{\left( {{{\log }_a}b - 1} \right)}^2}}}$

\(\dfrac{3}{2}\left( {{{\log }_a}b - 1} \right) + \dfrac{3}{2}\left( {{{\log }_a}b - 1} \right) + \dfrac{{12}}{{{{\left( {{{\log }_a}b - 1} \right)}^2}}}\)\( \ge 3.\,\sqrt[3]{{\dfrac{3}{2}\left( {{{\log }_a}b - 1} \right).\dfrac{3}{2}\left( {{{\log }_a}b - 1} \right).\dfrac{{12}}{{{{\left( {{{\log }_a}b - 1} \right)}^2}}}}}\)

\( \Leftrightarrow 3\left( {{{\log }_a}b - 1} \right) + \dfrac{{12}}{{{{\left( {{{\log }_a}b - 1} \right)}^2}}} \ge 9\) \(\left( {**} \right)\)

Từ \(\left( * \right)\)và \(\left( {**} \right)\) ta có \(P \ge 9\)

Dấu bằng xảy ra khi và chỉ khi \(\left\{ \begin{array}{l}b = \dfrac{1}{2}\\\dfrac{3}{2}\left( {{{\log }_a}b - 1} \right) = \dfrac{{12}}{{{{\left( {{{\log }_a}b - 1} \right)}^2}}}\end{array} \right.\) \( \Leftrightarrow \left\{ \begin{array}{l}b = \dfrac{1}{2}\\{\left( {{{\log }_a}b - 1} \right)^3} = 8\end{array} \right.\)

\( \Leftrightarrow \left\{ \begin{array}{l}b = \dfrac{1}{2}\\{\log _a}b - 1 = 2\end{array} \right.\)\( \Leftrightarrow \left\{ \begin{array}{l}b = \dfrac{1}{2}\\{\log _a}b = 3\end{array} \right.\)\( \Leftrightarrow \left\{ \begin{array}{l}b = \dfrac{1}{2}\\b = {a^3}\end{array} \right.\)\( \Leftrightarrow \left\{ \begin{array}{l}b = \dfrac{1}{2}\\a = \sqrt[3]{b} = \sqrt[3]{{\dfrac{1}{2}}}\end{array} \right.\)

Vậy \(\min P = 9\)

Câu 20 Trắc nghiệm

Với \(m\) là tham số thực dương khác $1$. Hãy tìm tập nghiệm \(S\) của bất phương trình

\({\log _m}(2{x^2} + x + 3) \le {\log _m}(3{x^2} - x)\). Biết rằng  \(x = 1\) là một nghiệm của bất phương trình.

Bạn đã chọn sai | Đáp án đúng: c
Bạn đã chọn đúng | Đáp án đúng: c
Bạn chưa làm câu này | Đáp án đúng: c

Điều kiện: \(\left\{ \begin{array}{l}2{x^2} + x + 3 > 0\\3{x^2} - x > 0\end{array} \right. \Leftrightarrow \left[ \begin{array}{l}x > \dfrac{1}{3}\\x < 0\end{array} \right.\)

Do \(x=1\) là một nghiệm của bất phương trình nên \({\log _m}(2.{1^2} + 1 + 3) \le {\log _m}(3.{1^2} - 1) \Leftrightarrow {\log _m}6 \le {\log _m}2 \Leftrightarrow 0<m < 1\)

Khi đó, ta có:

\({\log _m}(2{x^2} + x + 3) \le {\log _m}(3{x^2} - x)\)

\(\Leftrightarrow 2{x^2} + x + 3 \ge 3{x^2} - x\)\( \Leftrightarrow {x^2} - 2x - 3 \le 0 \Leftrightarrow  - 1 \le x \le 3\)

Kết hợp với điều kiện xác định ta có nghiệm của bpt là : \(S = \left[ { - 1;0} \right) \cup \left( {\dfrac{1}{3};3} \right)\)